1
IIT-JEE 2006
MCQ (Single Correct Answer)
+6
-1.5
Match the following : $$(3, 0)$$ is the pt. from which three normals are drawn to the parabola $${y^2} = 4x$$ which meet the parabola in the points $$P, Q $$ and $$R$$. Then

Column $${\rm I}$$
(A) Area of $$\Delta PQR$$
(B) Radius of circumcircle of $$\Delta PQR$$
(C) Centroid of $$\Delta PQR$$
(D) Circumcentre of $$\Delta PQR$$

Column $${\rm I}$$$${\rm I}$$
(p) $$2$$
(q) $$5/2$$
(r) $$(5/2, 0)$$
(s) $$(2/3, 0)$$

A
$$\left( A \right) - \left( p \right),\left( B \right) - \left( q \right),\left( C \right) - \left( s \right),\left( D \right) - \left( r \right)$$
B
$$\left( A \right) - \left( p \right),\left( B \right) - \left( q \right),\left( C \right) - \left( r \right),\left( D \right) - \left( s \right)$$
C
$$\left( A \right) - \left( s \right),\left( B \right) - \left( r \right),\left( C \right) - \left( p \right),\left( D \right) - \left( q \right)$$
D
$$\left( A \right) - \left( r \right),\left( B \right) - \left( s \right),\left( C \right) - \left( q \right),\left( D \right) - \left( p \right)$$
2
IIT-JEE 2006
MCQ (Single Correct Answer)
+3
-0.75
One angle of an isosceles $$\Delta $$ is $${120^ \circ }$$ and radius of its incircle $$ = \sqrt 3 $$. Then the area of the triangle in sq. units is
A
$$7 + 12\sqrt 3 $$
B
$$12 - 7\sqrt 3 $$
C
$$12 + 7\sqrt 3 $$
D
$$4\pi $$
3
IIT-JEE 2006
MCQ (More than One Correct Answer)
+5
-1.25
In $$\Delta ABC$$, internal angle bisector of $$\angle A$$ meets side $$BC$$ in $$D$$. $$DE \bot AD$$ meets $$AC$$ in $$E$$ and $$AB$$ in $$F$$. Then
A
$$AE$$ is $$HM$$ of $$b$$ and $$c$$
B
$$AD$$ $$ = {{2bc} \over {b + c}}\cos {A \over 2}$$
C
$$EF$$ $$ = {{4bc} \over {b + c}}\sin {A \over 2}$$
D
$$\Delta AEF$$ is isosceles
4
IIT-JEE 2006
Subjective
+6
-0
Match the following

Column $$I$$

(A) $$\sum\limits_{i = 1}^\infty {{{\tan }^{ - 1}}\left( {{1 \over {2{i^2}}}} \right) = t,} $$ then tan $$t=$$

(B) Sides $$a, b, c$$ of a triangle $$ABC$$ are in $$AP$$ and
$$\cos {\theta _1} = {a \over {b + c}},\,\cos {\theta _2} = {b \over {a + c}},\cos {\theta _3} = {c \over {a + b}},$$
then $${\tan ^2}\left( {{{{\theta _1}} \over 2}} \right) + {\tan ^2}\left( {{{{\theta _3}} \over 2}} \right) = $$

(C) A line is perpendicular to $$x + 2y + 2z = 0$$ and
passes through $$(0, 1, 0)$$. The perpendicular distance of this line from the origin is

Column $$II$$

(p) $$1$$

(q) $${{\sqrt 5 } \over 3}$$

(r) $${2 \over 3}$$

JEE Advanced Papers
EXAM MAP
Medical
NEET
Graduate Aptitude Test in Engineering
GATE CSEGATE ECEGATE EEGATE MEGATE CEGATE PIGATE IN
CBSE
Class 12